01PRA1:Kapitola4: Porovnání verzí

Z WikiSkripta FJFI ČVUT v Praze
Přejít na: navigace, hledání
Řádka 1: Řádka 1:
 
%\wikiskriptum{01PRA1}
 
%\wikiskriptum{01PRA1}
\chapter{Absolutně spojitá rozdělení}
+
\section{Absolutně spojitá rozdělení}
 
   
 
   
 
\begin{definition}[ASR, SASR]
 
\begin{definition}[ASR, SASR]

Verze z 1. 8. 2010, 16:50

PDF [ znovu generovat, výstup z překladu ] Kompletní WikiSkriptum včetně všech podkapitol.
PDF Této kapitoly [ znovu generovat, výstup z překladu ] Přeložení pouze této kaptioly.
ZIPKompletní zdrojový kód včetně obrázků.

Součásti dokumentu 01PRA1

součástakcepopisposlední editacesoubor
Hlavní dokument editovatHlavní stránka dokumentu 01PRA1Karel.brinda 4. 10. 201022:39
Řídící stránka editovatDefiniční stránka dokumentu a vložených obrázkůAdmin 7. 9. 201513:49
Header editovatHlavičkový souborKarel.brinda 8. 3. 201118:28 header.tex
Kapitola0 editovatPředmluvaAdmin 4. 8. 201009:45 predmluva.tex
Kapitola1 editovatMotivaceValapet2 5. 3. 201619:43 kapitola1.tex
Kapitola2 editovatAxiomatická definice pravděpodobnostiPitrazby 18. 2. 201200:46 kapitola2.tex
Kapitola3 editovatDiskrétní náhodné veličinySnilard 8. 3. 201100:55 kapitola3.tex
Kapitola4 editovatAbsolutně spojitá rozděleníPitrazby 18. 2. 201201:06 kapitola4.tex
Kapitola5 editovatCharakteristiky náhodných veličinJakub.flaska 1. 8. 201016:49 kapitola5.tex
Kapitola6 editovatLimitní věty teorie pravděpodobnostiPitrazby 18. 2. 201201:30 kapitola6.tex
Kapitola7 editovatStatistikaJakub.flaska 1. 8. 201017:22 kapitola7.tex

Vložené soubory

soubornázev souboru pro LaTeX
Soubor:01PRA1_kap1_Uloha_na_nedeli.pdf 01PRA1_kap1_Uloha_na_nedeli.pdf
Soubor:01PRA1_kap1_Buffonuv_problem.pdf 01PRA1_kap1_Buffonuv_problem.pdf
Soubor:01_PRA1_kap1_Bertranduv_paradox.pdf 01PRA1_kap1_Bertranduv_paradox.pdf

Zdrojový kód

%\wikiskriptum{01PRA1}
\section{Absolutně spojitá rozdělení}
 
\begin{definition}[ASR, SASR]
Buďte $ (X_1,\dots,X_n) $ náhodné veličiny. Říkáme že mají (pro $ n \geq 2 $ sdružené) rozdělení
absolutně spojitého typu (absolutně spojité rozdělení) - ASR/SASR - pokud na prostoru existuje borelovsky měřitelná funkce
\begin{equation}
f_{\mathbf{X}} : \mathbb{R}^n \to \mathbb{R}
\end{equation}
taková, že
\begin{equation}
\mathrm{F}_{\mathbf{X}}(\mathbf{x}) = \int_{-\infty}^{x_1} \int_{-\infty}^{x_2} \dots \int_{-\infty}^{x_n}
f_{\mathbf{X}}(\mathbf{t}) d\mathbf{t}\ \ \ \ \ \ \ \forall \mathbf{x} \in \mathbb{R}^n
\end{equation}
 
Funkci $ f_{\mathbf{X}}(\mathbf{t}) $ nazýváme (sdruženou) hustotou pravděpodobnosti (vzhledem k Lebesgueově míře) náhodné veličiny $ \mathbf{X} $.
\end{definition}
 
\begin{definition}[Absolutní spojitost]
Říkáme, že funkce $ F: \mathbb{R} \to \mathbb{R} $ je absolutně spojitá na intervalu $ (a,b) $,
pokud $(\forall \varepsilon > 0)(\exists \delta > 0)(\forall n)\left(\forall\left( a_j,b_j\right)
\subset \left(a,b\right)\right)$ \begin{equation}
\sum_{j=1}^{n}|a_j - b_j| < \delta \Rightarrow \sum_{j=1}^{n} |F(a_j) - F(b_j)| < \varepsilon
\end{equation}
\end{definition}
 
\begin{theorem}
Funkce $ \mathrm{F} : \mathbb{R} \to \mathbb{R} $ je absolutně spojitá, pokud existuje funkce $ f
: \mathbb{R} \to \mathbb{R} $ borelovsky měřitelná na $ \left(\Omega,\mathcal{B} \right) $ taková,
že
\begin{equation}
\mathrm{F}(x) = \int_{-\infty}^{x} f(t)\mathrm{d}t\ \ \ \ \ \forall \mathbf{t} \in \mathbb{R}
\end{equation}
V bodech spojistosti funkce $ f $ navíc platí $ \mathrm{F}'(x) = f(x) $
\end{theorem}
 
\begin{proof}
\ 
\begin{description}
\item[$ \Leftarrow $] { Důkaz pojmeme poněkud obecněji, a půjdeme na to přes borelovské množiny (jak
jinak...). Buď funkce $ f : \mathbb{R} \to \mathbb{R} $ borelovsky měřitelná z $
L\left(\mathbb{R}^n\right) $. Potom $ \left(\forall \varepsilon > 0\right) \left(\exists \delta
> 0 \right)\left(\forall M \in \mathcal{B}_n \right) $ platí
$$\lambda\left(M\right)<\delta \Rightarrow \left| \int_{M} f(t) \mathrm{d}t \right| \leq \varepsilon $$
tj. pro všechny borelovské množiny $ M $, které jsou podmnožinami $ \mathbb{R}^{n} $, platí jisté
tvrzení. Potom je ale integrál z funkce $ f $ funkcí absolutně spojitou. Funkce $ \nu(M) $, daná
předpisem $$ \nu (M) = \int_{M}f $$ je tedy absolutně spojitá.
 
Současně se však jedná o neurčitý integrál, a platí dokonce, že pokud $ f \geq 0 $, potom je $ \nu $
dokonce mírou na $ (\mathcal{R}^n,\mathcal{B}_n) $. }
\item[$ \Rightarrow $] { Uvažujme $ \Omega, \mathcal{A}, X, \mathrm{P} \to \mathrm{F}_X \to f_x $, a
nechť $ n \geq 1 $. Potom
$$ \mathrm{F}_x(x) = \int_{-\infty}^{x_1}\cdots\int_{-\infty}^{x_n} f_x(t)\mathrm{d}t = \int_{B =
\times_{j=1}^{n} (-\infty,x_j]} f_x(t) \mathrm{d}t = $$
$$ = \nu_{\mathrm{F}_X} \left( \times_{j=1}^{n} ( -\infty,x_j ] \right) $$
$$ \tau_n = \left\{ \times_{j=1}^{n} (-\infty,x_j]\ |\ x_j \in \mathcal{R} \right\} \subset
2^{\mathcal{R}^n} \Rightarrow \sigma(\tau_n) = \mathcal{B}_n $$
 
Potom ale můžeme $ \nu_{\mathrm{F}} $ jednoznačně rozšířit z $ \tau_n $ na $ \mathcal{B}_n $, a
navíc pokud $ f_{X} \geq 0 $, potom je $ \nu_{\mathrm{F}_X} $ míra na $
\left( \mathbb{R}^n,\mathcal{B}_n \right) $.}
\end{description}
\end{proof}
 
\begin{definition}[Absolutní spojitost míry vzhledem k míře]
Nechť $ \nu $ a  $ \lambda $ jsou míry na $ (\left( \mathbb{R}^n,\mathcal{B}_n \right) $. Říkáme,
že míra $ \nu $ je absoulutně spojitá vzhledem k míře $ \lambda $ (značíme $ \nu << \lambda $),
pokud $$ \lambda(B) = 0 \Rightarrow \nu(B) = 0\ \ \ \ \forall B \in \mathcal{B}_n $$
\end{definition}
 
\begin{definition}[$ \sigma $-konečná míra]
Míra $ \lambda $ se nazývá $ \sigma $-konečná, pokud existuje posloupnost $ \left( B_j
\right)_{1}^{\infty} \in \mathcal{B}_{n} $ taková, že
$$ \cup_{j=1}^{\infty}B_j = \mathbb{R}^n $$
$$ \lambda(B_j) < \infty $$
\end{definition}
 
\begin{note}
Lebesgueova míra $ \lambda $ je $ \sigma $-konečná. Stačí vzít například
$$ \mathbb{R} = \bigcup_{j=1}^{\infty} (-j,j) $$
přičemž zřejmě $ \lambda(B_j) = 2j < \infty $.
\end{note}
 
\begin{theorem}[Radon-Nikodymova]
\label{radau-nikodym}
Nechť jsou $ \nu $ a $ \lambda $ míry na $ (\mathbb{R}^n,\mathcal{B}_n) $ takové, že míra $ \lambda $ je
$ \sigma $-konečná, a přitom $ \nu << \lambda $. Potom existuje $ f \geq 0 $ borelovsky
měřitelná na $ (\mathbb{R}^n,\mathcal{B}_n) $, taková že
$$ \nu(B) = \int_{B} f \mathrm{d}\lambda \ \ \ \ \forall B \in \mathcal{B}_n $$
Funkce $ f $ je navíc dána jednoznačně (až na množinu míry nula vzhledem k míře $ \lambda $). To
znamená že pokud $ \nu(B) = \int_{B} g \mathrm{d} \lambda $, potom $ \forall B \in \mathcal{B}
\Rightarrow f = g $.
 
Funkce $ f $ se nazývá Radau-Nikodymova derivace míry $ \nu $ vzhledem k míře $ \lambda $, a značíme
ji
$$ f = \frac{d\nu}{d\lambda} $$
\end{theorem}
 
\begin{note}
Pokud položíme $ \nu = \mathrm{P} $, potom $$ f = \frac{d\mathrm{P}}{d\lambda} $$
je hustota pravděpodobnosti $ \mathrm{P} $ vzhledem k míře $ \lambda $.
\end{note}
 
\begin{note}[Lebesgueův rozklad distribuční funkce]
Pro libovolnou distribuční funkci $ \mathrm{F}_x $ platí
$$ \mathrm{F}_X(x) = A(x) + K(x) + S(x) $$
kde
\begin{description}
\item[$ A(x) $] { je absolutně spojitá funkce }
\item[$ K(x) $] { je skokovitá s nejvýše spočetně mnoha skoky }
\item[$ S(x) $] { je singulární spojitá část (roste jen na množině míry nula - růst musí, protože
se jedná o distribuční funkci)}
\end{description}
\end{note}
 
\begin{theorem}
Mějme náhodnou veličinu $ \mathbf{X} = (X_1,\dots,X_n) $ s SASR a hustotou pravděpodobnosti $ f_{\mathbf{X}} $. Potom $ \mathbf{X}' = (X_1,\dots,X_{j-1},X_{j+1},\dots,X_n) $ má také SASR, a platí
\begin{equation}
f_{\mathbf{X}'}(\mathbf{x}') = \int_{-\infty}^{+\infty} f_{\mathbf{X}}(\mathbf{x})dx_j\ \ \ \ \forall \mathbf{x}'
\end{equation}
\end{theorem}
 
\begin{proof}
\ 
$$ \mathrm{F}_{\mathbf{X}'}(\mathbf{x}') = \lim_{x_j \to +\infty} \mathrm{F}_{\mathbf{X}}(\mathbf{x}) = \lim_{x_j \to \infty} \int_{-\infty}^{x_1} \cdots \int_{-\infty}^{x_n} f_{\mathbf{X}}(t)d\mathbf{t} = \{ Fubini \} = $$
$$ = \int_{-\infty}^{x_1}\cdots\int_{-\infty}^{x_{j-1}} \int_{-\infty}^{x_{j+1}} \cdots
\int_{-\infty}^{x_n} \underbrace{\left( \int_{-\infty}^{+\infty}f_{\mathbf{X}}(\mathbf{t}) d\mathbf{t}_j
\right)}_{f_{\mathbf{X}'}(\mathbf{t}')} d\mathbf{t}' $$
 
$ f_{\mathbf{X}'}(\mathbf{x}') $ nazýváme marginální hustotou pravděpodobnosti náhodné veličiny $
\mathbf{X}' $.
\end{proof}
 
\begin{theorem}
Platí
\begin{equation}
f_{X_{i_1},X_{i_2},\dots,X_{i_k}}(x_{i_1},x_{i_2},\dots,x_{i_k}) =
\int_{-\infty}^{+\infty}dx_{i_{k+1}} \dots \int_{-\infty}^{+\infty}dx_{i_n}
f_{\mathbf{X}}(\mathbf{t})d\mathbf{t}
\end{equation}
kde $ (i_1,\dots,i_n) $ je permutace $ \widehat{n} $.
\end{theorem}
 
\begin{example}
Nechť $ (X,Y) $ mají SASR, přičemž
$$ f_{(X,Y)}(x,y) = \left\{ \matrix{\exp\left(-(x+y)\right) &\ \ & (x,y) \in \mathbb{R}_{+}^{2} \cr
0 &  & \textrm{jinde}} \right. $$
Nalezněte $ f_Y(y) $.
\end{example}
 
$$ f_Y(y) = \int_{-\infty}^{+\infty}f_{(x,Y)}(x,y) \mathrm{d}x $$
$$ f_Y(y) = \left\{ \matrix{0 &\ \ & y < 0 \cr \int_{0}^{+\infty} \exp(-(x+y))\mathrm{d}x =
\exp(-y)& & \textrm{jinak}} \right. $$
 
\begin{theorem}
Nechť $ \mathbf{X} = (X_1,\dots,X_n) $ mají SASR. Potom jsou $ X_1,\dots,X_n $ nezávislé, právě
když platí
\begin{equation}
f_{\mathbf{X}}(\mathbf{x}) = \prod_{j=1}^{n} f_{X_j}(x_j)\ \ \ \forall \mathbf{x} \in \mathbb{R}^n
\end{equation}
\end{theorem}
 
\begin{proof}
\ 
Na začátku si uvědomme, že platí
$$ \mathrm{F}_{\mathbf{X}}(\mathbf{x}) = \int_{-\infty}^{x_1} \cdots \int_{-\infty}^{x_n}
f_{\mathbf{X}}(\mathbf{t})d\mathbf{t} = \prod_{j=1}^{n} \int_{-\infty}^{x_j}
f_{x_j}(t_j)dt_j = \prod_{j=1}^{n}\mathrm{F}_{X_j}(x_j) $$
a odtud již snadno dokážeme oba směry implikace
\begin{description}
\item[$ \Leftarrow $] { Tento směr je zřejmý z předchozího tvrzení. }
\item[$ \Rightarrow $] { Protože jsou $ X_1,\dots,X_n $ nezávislé, platí
$$ \mathrm{F}_{\mathbf{X}}(\mathbf{X}) = \prod_{j=1}^{n} \mathrm{F}_{x_j}(x_j) $$ přičemž každá
marginální složka má SASR, takže
$$ \prod_{j=1}^{n} \mathrm{F}_{x_j}(x_j) = \prod_{j=1}^{n} \int_{-\infty}^{x_j} f_{X_j}(t_j)
\mathrm{d}t_j = \int_{-\infty}^{x_1} \cdots \int_{-\infty}^{x_n} \left( \prod_{j=1}^{n}
f_{X_j}(t_j)\right) d\mathbf{t} $$ }
\end{description}
\end{proof}
 
\begin{theorem}[Vlastnosti $ f_{\mathbf{X}} $]
Buď $ \mathbf{X} $ $ n- $rozměrná náhodná veličina $ (n \geq 1) $. Potom platí
\begin{enumerate}
\item $ f_{\mathbf{X}}(\mathbf{x}) \geq 0 $ skoro všude na $ \mathbb{R}^n $
\item $ \int_{\mathbb{R}^n} f_{\mathbf{X}}(\mathbf{t})d\mathbf{t} $
\item $ \left( \forall B \in \mathcal{B}_n \right)\left( \mathrm{P}\left(X \in B \right)\right) =
\int_{B} f_{\mathbf{X}}(\mathbf{t}) d\mathbf{t} $
\end{enumerate}
\end{theorem}
 
\begin{proof}
\ 
\begin{enumerate}
\item { OK, protože
$$ \mathrm{F}_{\mathbf{X}}(\mathbf{t}) = \int_{-\infty}^{x_1} \cdots \int_{-\infty}^{x_2}
f_{\mathbb{X}} $$
Distribuční funkce musí růst v každé složce, a tak musí být $ f_{\mathbf{X}} $ nezáporná (až na
množinu míry nula). }
\item { OK, protože (i díky předchozímu bodu) víme, že
$$ \mathrm{F}_{\mathbf{X}}(+\infty,\dots,+\infty) = 1 $$ }
\item {
\begin{enumerate}
\item{ Platí
$$ \mathrm{P}\left(\mathbf{X} \in B \right) = \mathrm{P}\left( \mathbf{X}^{-1}\left(B\right) \right) = \left(\mathrm{P} \circ \mathbf{X}\right)^{-1}\left( B
\right) = \mathrm{P}^{\mathbf{X}} \left( B \right)  = \nu_{\mathrm{F}_{\mathbf{X}}} \left( B
\right) $$
$ \mathrm{P}^{\mathbf{X}}\left(B\right) $ je opět pravděpodobnostní míra, takže na ni můžeme použít
R.-N. větu (\ref{radau-nikodym}) s volbou $ \nu = \mathrm{P}^{\mathbf{X}} $. Potom dle R.-N. věty
existuje funkce $ f_{\mathbf{X}} $ taková, že $$\left(\forall B \in \mathcal{B}_n \right) \left(
\mathrm{P}_{\mathbb{X}}\left( B \right) = \int_{B} f_{\mathbb{X}} \right) $$ }
\item{
$$ \mathrm{P}\left( a < X \leq b \right) = \left( \int_{a}^{b} f_{X}(t)\mathrm{d}t \right) $$
$$ \mathrm{P}\left( \cap_{j=1}^{+\infty} \left\{ a_j < X_j \leq b_j \right\} \right) =
\int_{a_1}^{b_1} \cdots \int_{a_j}^{b_j} \cdots \int_{a_n}^{b_n} f_{\mathbb{X}}(\mathbf{t})
\mathrm{d}\mathbf{t} $$
$$ a_j \in \mathcal{R} \cup \{+\infty\} $$
$$ b_j \in \mathcal{R} \cup \{-\infty\} $$
}
\end{enumerate}
}
\end{enumerate}
\end{proof}
 
\begin{definition}[Podmíněná distribuční funkce]
Nechť jsou $ X $, $ Y $ náhodné veličiny. Potom podmíněnou distribuční funkcí náhodné veličiny $ X $
při dané hodnotě $ Y = y \in R_{Y} $ definujeme jako
$$ \mathrm{F}_{X|Y} = \lim_{\varepsilon \to 0+} \mathrm{P}\left( X \leq x\ |\ y - \varepsilon < Y \leq y +
\varepsilon \right) $$
za předpokladu, že limita existuje. Pokud navíc existuje funkce $ f_{X|Y}(x|y) \geq 0 $ taková, že
$$ \mathrm{F}_{X|Y}(x,y) = \int_{-\infty}^{x} f_{X|Y}(t|y)\mathrm{d}t\ \ \ \ \forall t \in
\mathcal{R} $$
potom ji nazýváme hustotou náhodné veličiny $ X $ podmíněnou $ Y = y $.
\end{definition}
 
\begin{lemma}
Nechť je náhodná veličina $ X $ absolutně spojitá (má ASR), tj. existují $ \left(
\mathrm{F}_{X}, f_{X} \right) $. Nechť je dále $ f_{X} $ spojitá v $ x_0 $. Potom
$$ \lim_{\varepsilon \to 0+} \frac{1}{2\varepsilon} \int_{x_0 - \varepsilon}^{x_0 + \varepsilon}
f_{X}(t)\mathrm{d}t = f_{x}(x_0) $$
\end{lemma}
 
\begin{proof}
\ 
$$ \frac{1}{2\varepsilon} \int_{x_0 - \varepsilon}^{x_0 + \varepsilon} f_{X} = \frac{1}{2\varepsilon}
\mathrm{P}\left(x_0 - \varepsilon < X \leq x_0 + \varepsilon \right) = \frac{1}{2\varepsilon} \left[
\mathrm{F}_X(x_0 + \varepsilon) - \mathrm{F}_X(x_0 - \varepsilon) \right] $$
a to již konverguje k $ {\mathrm{F}'}_X(x_0) = f_X(x_0) $.
\end{proof}
 
\begin{theorem}
Nechť veličiny $ X $ a $ Y $ mají SASR a nechť $ y_0 \in \mathcal{R}_Y $. Nechť dále platí
\begin{enumerate}
\item { $ f_{X,Y}(x,y) $ je spojitá v $ y_0 $ pro skoro všechna $ x $ }
\item { $ f_Y(y) $ je spojitá v $ y_0 $ a přitom $ f_y(y_0) > 0 $ }
\end{enumerate}
Potom 
$$ \exists f_{X|Y}(x,y_0) = \frac{f_{X,Y}(x,y_0)}{f_Y(y_0)} $$ pro skoro všechna $ X $.
\end{theorem}
 
\begin{proof}
\ 
$$ \mathrm{F}_{X|Y} = \lim_{\varepsilon \to 0+} \frac{\mathrm{P}\left( X \leq x, y_0 - \varepsilon < Y
\leq y_0 + \varepsilon \right)}{\mathrm{P}\left( y_0 - \varepsilon < Y \leq y_0 + \varepsilon \right)} = $$
$$ = \lim_{\varepsilon \to 0+} \frac{\frac{1}{2\varepsilon}\int_{-\infty}^{x}\mathrm{d}u \int_{y_0 -
\varepsilon}^{y_0 + \varepsilon} \mathrm{d}v f_{X,Y}(u,v) }{\frac{1}{2\varepsilon} \int_{y_0 -
\varepsilon}^{y_0 + \varepsilon}f_Y(y)\mathrm{d}y } = $$
$$ = {\int_{-\infty}^{x}\left( \lim_{\varepsilon \to +\infty} \frac{1}{2\varepsilon} \int_{y_0 -
\varepsilon}^{y_0 + \varepsilon} f_{X,Y}\mathrm{d}v \right)\mathrm{d}u }^{} \stackrel{lemma}{=}
\int_{-\infty}^{x} \underbrace{\frac{f_{X,Y}(u,y_0)}{f_Y(y_0) }}_{f_{X|Y}(u|y_0) } \mathrm{d}u $$
\end{proof}
 
\begin{theorem}
Nechť $ \mathbf{X} $ má SASR, a $ g : \mathbb{R}^n \to \mathbb{R}^m $ buď borelovsky měřitelná, přičemž $ m \leq n $. Potom $ \mathbf{Y} = g(\mathbf{X}) $ má také SASR a platí
\begin{equation}
f_{\mathbf{Y}}(\mathbf{y}) = \frac{\partial^m}{\partial y_1 \dots \partial y_m} \left(\int_{B_{\mathbf{y}}} f_{\mathbf{X}}(\mathbf{x}) d\mathbf{x} \right)
\end{equation}
kde $ B_{\mathbf{y}} = \left\{ \mathbf{x} \in \mathbb{R}^{n}\ |\ g(\mathbf{x}) \leq \mathbf{y} \right\} $ za předpokladu, že derivace existuje skoro všude vzhledem k $ \lambda $. Pokud $ m = n $ a $ g $ je navíc regulární a prosté zobrazení na otevřené množině $ G $, pro kterou platí $ \int_{G} f_{\mathbf{X}} = 1 $, potom 
\begin{equation}
f_{\mathbf{Y}}(\mathbf{y}) = \left\{\matrix{f_{\mathbf{X}}\left(g^{-1}\left(\mathbf{y}\right)\right)\left|\mathbb{J}_{g^{-1}}\left(\mathbf{y}\right)\right| & \textrm{\ pro\ } \mathbf{y} \in g(G) \cr 0 & \textrm{\ jinak}}\right.
\end{equation}
\end{theorem}
 
\begin{proof}
\ 
$$ F_{\mathbf{Y}}\left( \mathbf{y} \right) = \mathrm{P}\big( \underbrace{\mathbf{Y} \leq \mathbf{y}}_{\mathrm{po\ složkách}} \big) = \mathrm{P}\left( g\left(\mathbf{X}\right) \leq \mathbf{y} \right) = \mathrm{P}\left( \omega\ |\ \mathbf{X} \in B_{\mathbf{y}} \right) = \int_{B_{\mathbf{y}}} f_{\mathbf{X}} $$
takže platí
$$ f_{\mathbf{Y}}\left(\mathbf{y}\right) = \frac{\partial}{\partial y_1 \dots \partial y_m} \left(F_{\mathbf{Y}}(\mathbf{y}) \right) $$
Buď nyní $ \varphi : \mathbb{R}^{n} \to \mathbb{R}^{m} $ regulární a prosté zobrazení, $ K $ buď otevřená množina.  Potom 
$$ \int_{\varphi^{-1}(K)} f\left(\mathbf{X}\right)d\mathbf{x} = \int_{K}f\left(\varphi(\mathbf{t})\right)\left| \mathbb{J}_{\varphi(\mathbf{t})}\right|d\mathbf{t} $$
a nyní položme $ \varphi^{-1} = g $. Potom triviálně platí
$$ \int_{B_{\mathbf{y}}} f_{\mathbf{X}} d\mathbf{x} \stackrel{\mathrm{v.\ o\ subst.}}{=} \int_{-\infty}^{y_1}\dots\int_{-\infty}^{y_m} \underbrace{f_{\mathbf{X}}\left(g^{-1}(\mathbf{y}\right)\left|\mathbb{J}_{g^{-1}}(\mathbf{y}) \right|}_{f_{\mathbf{Y}}\left(\mathbf{y}\right)}d\mathbf{y} $$
\end{proof}
 
Specielním případem předchozí věty je $ m = n = 1,\ g \in C^{(1)},\ g'(x) \neq 0,\ g$ ryze monotonní. Potom platí
$$ f_{Y}\left(y\right) = f_{X}\left(g^{-1}(y)\right)\left|g^{-1}(y)\right| $$
 
\begin{theorem}
Buď $ g \in C^{(1)},\ g' \neq 0 $ po částech ryze monotonní, a nechť $ Y = g(X) $. Pokud $ g^{-1}\left(y\right) \neq \emptyset $, potom ve všech bodech $ t \in g^{-1}(y) $ platí
\begin{equation}
f_{Y}(y) = \sum_{t \in g^{-1}(y)} \frac{f_{X}(t)}{\left|g'(t)\right|}
\end{equation}
\end{theorem}
 
\begin{proof}
\ 
$$ \mathrm{F}_{Y}(y) = \mathrm{P}\left(Y \leq y\right) = \mathrm{P}\left( g(X) \leq y \right) = \mathrm{P}\left( X \in \cup_{i=1}^{m}I_{k}(y)\right) = \sum_{k=1}^{m} \mathrm{P} \left( X \in I_{k}(y) \right) = $$
$$ = \sum_{k=1}^{m}\mathrm{P}\left( a_k(y) < x \leq b_k(y) \right) = \sum_{k=1}^{m}\left[\mathrm{F}_{X}\left(b_k(y)\right) - \mathrm{F}_{X}\left(a_k(y)\right) \right]$$
$$ f_{Y}(y) = \sum_{k=1}^{m} \left( f_X\left(b_k(y)\right){b'}_k(y) - f_{X}\left(a_k(y)\right){a'}_k(y) \right)  = \sum_{t \in g^{-1}(y)} \frac{f_{X}(t)}{\left|g'(t)\right|}$$
kde $ a_k $ je klesající a $ b_k $ je rostoucí.
\end{proof}
 
\begin{theorem}
Buď $ \mathbf{X} = \left(X_1,\dots,X_n \right) $ nezávislé náhodné veličiny, z nich každá má ASR. Potom $ X_1 + \cdots + X_r = Y_1 $ a $ X_{r+1} + \cdots + X_{n} = Y_2 $ jsou nezávislé náhodné veličiny (předopkládáme $ 1 < r < n $).
\end{theorem}
 
\begin{proof}
\ 
$$ f_{\mathbf{X}} = \Pi_{j=1}^{n} f_{X_{j}}\ \ \Rightarrow\ \  f_{Y_{1},Y_{2}} = f_{Y_1}\cdot f_{Y_2} $$
\end{proof}
 
\subsection{Příklady SASR rozdělení}
\subsubsection{Gamma rozdělení $\ \ \ Gamma(\alpha,\beta) $}
$$ X \sim f_X(x) = \frac{1}{\Gamma(\alpha)} \frac{1}{\beta^{\alpha}} x^{\alpha-1}e^{-\frac{x}{\beta}}\ \ \textrm{\ pro\ } \alpha,\beta,x > 0 $$
$$ \mathrm{F}_{X}(x) = \int_{-\infty}^{x} \frac{1}{\Gamma(\alpha)} \frac{1}{\beta^{\alpha}} t^{\alpha-1}e^{-\frac{t}{\beta}} dt \ \ \ \textrm{(tzv. neúplná } \Gamma \textrm{\ funkce)} $$
 
\subsubsection{Beta rozdělení $\ \ \ Beta(p,q) $}
$$ X \sim f_X(x) = \frac{1}{B(p,q)}x^{p-1}(1-x)^{q-1}\ \ \textrm{\ pro\ } p,q > 0, x \in (0,1) $$
$$ \mathrm{F}_{X}(x) = \int_{-\infty}^{x} \frac{1}{B(p,q)}t^{p-1}(1-t)^{q-1} dt \ \ \ \textrm{(tzv. neúplná\ } B \textrm{ funkce)} $$
 
\subsubsection{Rovnoměrné rozdělení $\ \ \ U(G) $ }
Nechť $ G \subset \mathbb{R}^n $ je oblast, potom
$$ X \sim f_{X}(x) = \left\{ \matrix{\frac{1}{\mu\left(G\right)} & \textrm{\ pro\ } x \in G \cr 0 & \textrm{jinak}}  \right. $$
Specielně pro interval $ (a,b) $ můžeme například definovat $ \mu(G) = b - a $
 
\subsubsection{Exponenciální rozdělení $ \ \ \ Exp(\theta,\mu) $}
(Jedná se vlastně o specielní případ rozdělení Gaussova).
$$ X \sim f_{X}(x) = \frac{1}{\theta}e^{-\frac{x - \mu}{\theta}}\ \ \ \textrm{pro\ } x > \mu, \mu \in \mathbb{R}, \theta > 0 $$
$$ Exp(\theta,0) = Exp(\theta) = Gamma(1,\theta) $$
$$ \mathrm{F}_{X}(x) = 1 - e^{-\frac{x - \mu}{\theta}} $$
Používá se v Poissonově procesu, řídkých jevech ($ X_t $ - počet událostí, které nastaly v časovém intervalu $ [0,t] $).
 
\begin{theorem}
\label{exp-sum}
Buďte $ X_1,\dots,X_n $ i.i.d. (identically and independently distributed - nezávisle a stejně rozdělené) dle $ Exp(\theta) $. Potom $ X_1 + \cdots + X_n \sim Gamma(n,\theta) $.
\end{theorem}
 
\begin{proof}
\leftskip = 0.25 in\ \\
Nechť $ Y_1 = \sum_{j=1}^{n} X_j $, potom tedy $$ f_Y = f_{\mathbf{X}}\left( g^{-1}(\mathbf{y}) \right)\left| \mathbf{J}_{g^{-1}(\mathbf{y})}\right| $$
přičemž ale
$$ f_{\mathbf{X}}(\mathbf{x}) = \prod_{j=1}^{n} f_{X_j}(x_j) = \frac{1}{{\theta}^n}e^{-\frac{\sum_{j=1}^{n}x_j}{\theta}} $$
Definujme nyní $ n- $rozměrnou prostorovou transformaci $ g $, kterou poté využijeme při přechodu k hustotě $ f_Y $
$$ \begin{array}{ccl}
Y_1 & = & X_1 + X_2 + \cdots + X_n \\
Y_2 & = & X_2  \\
\vdots & & \vdots \\
Y_n & = & X_n
\end{array} $$
Inverzi nám stačí spočítat pouze pro složku $ X_1 $, inverze pro ostatní složky jsou dány primitivně. Transformace $ g^{-1} $ je tedy dána předpisem
$$ \begin{array}{ccl}
X_1 & = & Y_1 - Y_2 + \cdots + Y_n \\
X_2 & = & Y_2  \\
\vdots & & \vdots \\
X_n & = & Y_n
\end{array} $$
a tedy
$$ \mathbf{J}_{g^{-1}} = \left|\matrix{
\ \ 1 & -1 & -1 & \cdots & -1 \cr
0 & 1 & 0 & \cdots & 0 \cr
0 & 0 & \ddots & & \vdots \cr
0 & \vdots & & 1 & 0 \cr
0 & 0 & \cdots & 0 & 1 }
\right| = 1 $$
a platí tedy
$$ f_{\mathbf{Y}}(\mathbf{y}) = \left\{ \matrix{\frac{1}{{\theta}^{n}}e^{-\frac{y_1}{\theta}} & \textrm{\ pro\ } y_1,y_2,\dots,y_n > 0 \cr 0 & \textrm{\ jinak}} \right. $$
$$ f_{Y_1}(y_1) = \underbrace{\int \cdots \int}_{G} \frac{1}{{\theta}^{n}} e^{-\frac{y_1}{\theta}}dy_2 dy_3 \dots dy_n = $$
$$ \textrm{\ kde\ } G = \left\{\mathbf{y}\ \left|\matrix{y_1 - y_2 - \cdots - y_n > 0 \cr y_j > 0 \textrm{\ pro\ } j \in \widehat{n}}\right. \right\} $$
$$ = \frac{1}{{\theta}^{n}} e^{-\frac{y_1}{\theta}} \underbrace{\int \cdots \int}_{G} dy_3 dy_3 \dots dy_n  = \frac{1}{{\theta}^{n}}e^{-\frac{y_1}{\theta}}\frac{y_1^{n-1}}{(n-1)!} $$ 
\end{proof}
 
\begin{theorem}[Reprodukční vlastnost Gamma rozdělení]
Buďte $ X_1,\dots,X_n $ i.i.d. náhodné veličiny, takové že $ X_j \sim Gamma(\alpha_j,\beta) $. Potom platí
$$ \sum_{j=1}^{n}X_j \sim Gamma\left(\sum_{j=1}^{n}\alpha_j,\beta\right) $$
\end{theorem}
 
\begin{proof}
Pro $ \alpha_j = 1 $ je důsledkem věty \ref{exp-sum}, jinak vyplývá z momentové vytvářející funkce. 
\end{proof}
 
\subsubsection{Normální (Gaussovo) rozdělení $\ \ N(\mu,\sigma^{2}) $}
$$ X \sim f_{X}(x) = \frac{1}{\sqrt{2\pi}\sigma} e^{-\frac{(x-\mu)^{2}}{2\sigma^{2}}}\ \ \textrm{\ pro\ } x,\mu \in \mathbb{R}, \sigma > 0 $$
Nechť $ \sigma = 0, \mu  = 1 $, potom rozdělení $ N(0,1) $ označujeme jako standardní normální rozdělení, jeho hustotu pravděpodobnosti označujeme $ \varphi(x) $, distribuční funkci označujeme $ \Phi(x) $, tj.
\begin{equation}
\varphi(x) = \frac{1}{\sqrt{2\pi}}e^{-\frac{x^2}{2}}
\end{equation}
\begin{equation}
\Phi(x) = \frac{1}{\sqrt{2\pi}} \int_{-\infty}^{x}e^{-\frac{t^2}{2}}dt
\end{equation}
 
\begin{theorem}[Vlastnosti standardního normálního rozdělení]
\ 
\begin{enumerate}
\item $ \Phi(x) = 1 - \Phi(-x) $
\item $ X \sim N(\mu,\sigma^2)\ \ \Rightarrow\ \ \mathrm{F}_{X}(x) = \Phi\left(\frac{x - \mu}{\sigma} \right) $
\item $ \mathrm{P}\left(a < X \leq b \right) = \Phi\left(\frac{b - \mu}{\sigma}\right) - \Phi\left(\frac{a - \mu}{\sigma}\right) = \int_{\frac{a - \mu}{sigma}}^{\frac{b - \mu}{\sigma}} \frac{1}{\sqrt{2\pi}}e^{-\frac{t^2}{2}}dt $
\item $ X \sim N(\mu,\sigma^2)\ \ \Rightarrow\ \ aX + b \sim N(a\mu + b,a^2\sigma^2) $
\item $ X \sim N(\mu,\sigma^2)\ \ \Rightarrow\ \ \frac{X - \mu}{\sigma} \sim N(0,1) $
\item Nechť $ \forall j \in \widehat{n}\ \ X_j \sim N(\mu_j,\sigma_j^2)  $, nezávislé náhodné veličiny, nechť $ a_j \in \mathbb{R},j \in \widehat{n} $ a navíc nechť $ \exists k \in \widehat{n}$ tak, že $ a_k \neq 0 $. Potom
$$ \sum_{j=1}^{n}a_j X_j \sim N\left(\sum_{j=1}^{n}a_j \mu_j, \sum_{j=1}^{n}a_j^2\sigma_j^2 \right) $$
\item Buďte $ X_j,\ j\in \widehat{n} $ i.i.d. nezávislé náhodné veličiny s rozdělením $ N(\mu,\sigma^2) $. Potom
$$ \overline{X_n} = \frac{1}{n}\sum_{j=1}^{n}X_j \sim N\left(\mu,\frac{\sigma^2}{n}\right) $$
$$ \sqrt{n}\frac{\left( \overline{X_n} - \mu \right)}{\sigma} \sim N(0,1) $$
\item $ X \sim N(0,1)\ \ \Rightarrow\ \ X^2 \sim Gamma\left(\frac{1}{2},2\right) = \chi^{2}_{(1)}$
\item $ X \sim N(0,1)\ \ \Rightarrow\ \ Y = e^x $ má tzv. lognormální rozdělení $ LN(\mu,\sigma^2) $ hustotou 
$$ f_Y(y) = \left\{\matrix{\frac{1}{y}f_X(\ln y) = \frac{1}{\sigma y} \varphi\left( \frac{\ln y - \mu}{\sigma}\right) & \textrm{\ pro\ } y > 0 \cr 0 & \textrm{\ pro\ } y \leq 0} \right. $$
\item Buďte $ X,Y \sim N(0,1) $ a nezávislé (tj. i.i.d.). Potom $ U = \frac{X}{Y} $ má Cauchyovo rozdělení s hustotou 
$$ f_U(u) = \frac{1}{\pi(1+u)^2}\ \ \ \textrm{\ pro každé\ } u \in \mathbb{R} $$
\end{enumerate}
\end{theorem}
 
\begin{proof}
\ \begin{enumerate}
\item Triviální. Stačí provést substituci $ x \to (-x) $.
\item $$ \mathrm{F}_X(x) = \frac{1}{\sqrt{2\pi}\sigma} \int_{-\infty}^{x} e^{-\frac{(t-\mu)^{2}}{2\sigma^2}}dt = \left| \textrm{\ substituce\ }\frac{t - \mu}{\sigma} = z \right| = $$
$$ = \frac{1}{\sqrt{2\pi}\sigma}\int_{-\infty}^{\frac{(x-\mu)^2}{\sigma}}e^{\left( \frac{-z^2}{2} \right)} dz = \Phi\left(\frac{x-\mu}{\sigma} \right) $$
\item Vyplývá z (2), protože
$$ \mathrm{P}\left( a < X \leq b \right) = \mathrm{F}_{X}(b) - \mathrm{F}_{X}(a - 0) = \Phi\left(\frac{b - \mu}{\sigma} \right) - \Phi\left(\frac{a - \mu}{\sigma}\right) = \cdots $$
\item Buď $ X \sim N\left(\mu,\sigma^2\right) $, a nechť $ Y = aX + b,\ a \neq 0 $. Potom
$$ f_Y(y) = \frac{1}{|a|} f_X\left( \frac{y - b}{a} \right) = \frac{1}{|a|\sqrt{2\pi}\sigma} e^{-\frac{1}{2\sigma^2}\left( \frac{y-b}{a} - \mu \right)^{2}} = $$
$$ = \frac{1}{\sqrt{2\pi}|a|\sigma}e^{-\frac{1}{2a^2\sigma^2}\left(y - \left(a\mu + b\right)\right)^2} \sim N\left(\mu',\sigma'^2\right) $$
\item Je přímým důsledkem (4), protože pro $ a = \frac{1}{\sigma},\ b=-\frac{\mu}{\sigma} $ platí
$$ \frac{X - \mu}{\sigma} \sim N(0,1) $$
\item Důkaz provedeme indukcí.
\begin{description}
\item [$ n = 2 $] { tj. chceme dokázat, že $ X_1 + X_2 \sim N\left(\mu_1 + \mu_2, \sigma_1^2 + \sigma_2^2 \right) $ 
$$ Y = X_1 + X_2\ \ \Rightarrow\ \  f_Y(y) = \int_{-\infty}^{\infty}f_{X_1}(v)f_{X_2}(y-v)dv = \ \ \ \textrm{tzv. konvoluce}  $$
$$ = \frac{1}{\left(\sqrt{2\pi}\right)^2 \sigma_1 \sigma_2} \int_{-\infty}^{\infty} e^{\frac{(v - \mu_1)^2}{2\sigma_1^2}} e^{-\frac{(y - v - \mu_2)^2}{2 \sigma^2_2}}dv $$
přitom obecně platí
$$ \int_{-\infty}^{\infty} e^{(-av^2 + bv + c)}dv = e^{\left(c + \frac{b^2}{4a^2}\right)} \int_{-\infty}^{\infty} e^{-a \left( v - \frac{b}{2a}\right)^2 }dv = \left| \textrm{\ substituce\ }\right| = $$
$$ = e^{\left(c + \frac{b^2}{4a^2}\right)}\int_{-\infty}^{\infty}e^{-at^2}dt = \sqrt{\frac{\pi}{a}}e^{\left( c + \frac{b^2}{4a^2} \right)} $$
polože nyní
$$ a = \frac{1}{2\sigma_1^2} + \frac{1}{\sigma_2^2} $$
$$ b = \frac{\mu_1}{\sigma_1^2} + \frac{y - \mu_2}{\sigma_2^{2}} $$
$$ c = \frac{-\mu_1^2}{2\sigma^2_1} - \frac{(y - \mu_2)^2}{2\sigma_2^2} $$
potom zřejmě
$$ \sqrt{\frac{\pi}{a}}e^{\left( c + \frac{b^{2}}{4a^2} \right)} = \frac{1}{\sqrt{2\pi}\sqrt{\sigma_1^2 + \sigma_2^2}}e^{-\frac{(y - \mu_1 - \mu_2)^2}{2(\mu_1^2 + \mu_2^2)}} $$
}
\item [$ n \to n+1 $] {
$$ \sum_{j=1}^{n+1} X_j = \sum_{j=1}^{n} X_j + X_{n+1} \sim N\left( \sum_{j=1}^{n+1}\mu_j,\sum_{j=1}^{n+1}\sigma_j^2 \right) $$
}
\end{description}
\item Plyne z (6), stačí volit $ a = \frac{1}{n},\ \mu_j = \mu,\ \sigma_j^2 = \sigma^2 $. Potom
$$ \underbrace{\frac{1}{n}\sum_{j=1}^{n} X_j}_{\overline{X_n}} \stackrel{(6)}{\sim} N\left( \sum_{j=1}^{n} \frac{1}{n} \mu, \sum_{j=1}^{n}\frac{1}{n^2} \sigma^{2} \right) = N \left( \mu, \frac{\sigma^2}{n} \right)$$
To znamená, že pokud uděláme aritmetický průměr ($ \overline{X_n} $) z i.i.d. náhodných veličin, přiblížíme se střední hodnotě (tj. zmenší se odchylka). Druhá část tvrzení vyplývá přímo z vlastnosti (5).
\end{enumerate}
\end{proof}
 
\begin{example}
Nechť $ X \sim f_X(x) = \frac{1}{\sqrt{2\pi}\sigma}e^{-\frac{(x-\mu)^2}{2\sigma^2}} $, $ \sigma > 0 $, tj. $ X \sim N(\mu, \sigma^2) $. Potom
$$ \int_{a}^{b}f_X(t)dt = \mathrm{F}_X(b) - \mathrm{F}_X(a) = \mathrm{P}\left(a < X \leq b\right) $$
$$ \mathrm{P}\left(\mu - \sigma < X \leq \mu + \sigma \right) = \mathrm{P}\left(\mu - \sigma \leq X \leq \mu + \sigma \right)\ \ \textrm{(díky spojitosti)} $$
takže
$$ \mathrm{P}\left(\mu - \sigma \leq X \leq \mu + \sigma \right) = \Phi\left(1\right) - \underbrace{\Phi\left(-1\right)}_{1 - \Phi(1)} \doteq 0.6826 $$
$$ \mathrm{P}\left(\mu - 2\sigma \leq X \leq \mu + 2\sigma \right) = \Phi(2) - \Phi(-2) \doteq 0.9545 $$
$$ \mathrm{P}\left(\mu - 3\sigma \leq X \leq \mu + 3\sigma \right) = \Phi(3) - \Phi(-3) \doteq 0.9973 $$
\end{example}
 
\begin{theorem}[Pearsonovo rozdělení $ \chi^2 $]
Nechť $ X_1,\dots,X_n $ jsou i.i.d. veličiny s rozdělením $ N(0,1) $. Pak $ X^2 = \sum_{j=1}^{n}X_j^2 $ má rozdělení $ \chi^2 $ s $ n $ stupni volnosti (značíme $ \chi^2(n) $) s hustotou
\begin{equation}
f_{\chi^2}(y) = \left\{ \matrix{\frac{1}{2\Gamma\left(\frac{n}{2}\right)}\left(\frac{y}{2}\right)^{\frac{n}{2}-1}e^{-\frac{y}{2}} & \textrm{\ pro\ } y > 0 \cr 0 & \textrm{\ jinak}} \right.
\end{equation}
\end{theorem}
 
\begin{proof}
\ \\
V zásadě máme dvě možnosti. Buď se můžeme mořit s transformací hustot $ \chi^2 = g(\mathbf{X}) $, nebo na to můžeme jít fikaně přes reprodukční vlastnost Gamma rozdělení. A my fikaní jsme, a navíc víme, že
$$ X_j \sim N(0,1)\ \ \Rightarrow\ \ X_j^2 \sim Gamma\left(\frac{1}{2},2\right) $$
takže snadno z reprodukční vlastnosti Gamma rozdělení ukážeme, že platí
$$ \sum_{j=1}^{n}X_j^2 \sim Gamma\left(\frac{n}{2},2\right) $$
\end{proof}
 
\subsubsection{Studentovo rozdělení}
\begin{theorem}[Studentovo rozdělení]
Buďte $ X,Y $ takové náhodné veličiny, že $ X \sim N(0,1) $, $ Y \sim \chi^2(n) $. Potom náhodná veličina
\begin{equation}
T = \frac{X}{\sqrt{\frac{Y}{n}}}
\end{equation}\textbf{Studentovo rozdělení} $ t(n) $ s $ n $ stupni volnosti a s hustotou
\begin{equation}
f_T(t) = \frac{1}{B\left(\frac{n}{2},\frac{1}{2}\right)}n^{\frac{n}{2}}(n+t^2)^{-\frac{n+1}{2}}\ \ \ \textrm{pro každé\ } t \in \mathbb{R}
\end{equation}
\end{theorem}
 
\begin{proof}
\ \\
Nechť $ Z = \sqrt{\frac{Y}{n}} $, potom $ g^{-1} : y = uz^2 $, a tedy
$$ f_Z(z) = 2nz f_Y(uz^2)\ \ \ \textrm{pro každé\ } z > 0 $$
$$ T = \frac{X}{Z} = \left| \matrix{\textrm{dle vztahu} \cr \textrm{pro podíl}} \right| = f_T(t) = \int_{0}^{\infty} z f_X(zt)f_Z(z)dz = $$
$$ = \int_{0}^{\infty} z \frac{1}{\sqrt{2\pi}}e^{-\frac{z^2t^2}{2}} (2nz) \frac{1}{2\Gamma \left(\frac{n}{2}\right)}\left(\frac{nz^2}{2}\right)^{\frac{n}{2}-1}e^{-\frac{nz^2}{2}} dz = $$
$$ = \frac{1}{\sqrt{2\pi}}\left(\frac{n}{2}\right)^{\frac{n}{2}} \frac{1}{\Gamma\left(\frac{n}{2}\right)} \int_{0}^{\infty}z^n e^{-\frac{z^2(t^2+n)}{2}}dz = \left| \textrm{\ substituce\ } \matrix{\frac{z^2(t^2 + n)}{2} = x \cr dz = \frac{1}{\sqrt{t^2 + n}}x^{-\frac{1}{2}}dx } \right| = $$
$$ = \frac{2}{\sqrt{\pi}} \left(\frac{n}{2}\right) $$
\end{proof}
 
\subsubsection{Fischerovo rozdělení}
\begin{theorem}[Fischerovo rozdělení]
Buďte $ X,Y $ nezávislé náhodné veličiny, takové že $ X \sim \chi^2(m) $, $ \chi^2(n) $. Potom náhodná veličina
\begin{equation}
\frac{X/m}{Y/n}
\end{equation}
má tzv. \textbf{Fischerovo rozdělení} $ F(m,n) $ se dvěma stupni volnosti $ (m,n) $, a s hustotou
\begin{equation}
f_F(u) = \frac{1}{B\left(\frac{m}{2},\frac{n}{2}\right)}\left(\frac{m}{n}\right)^{\frac{m}{2}}u^{\frac{m}{2}-1}\left( 1  +\frac{m}{n}u \right)^{-\frac{m+n}{2}}\ \ \ \textrm{\ pro každé\ } u > 0
\end{equation}
\end{theorem}